The purpose of economic systems is

Answers

Answer 1
An economic system is a means by which societies or governments organize and distribute available resources, services, and goods across a geographic region or country. Economic systems regulate factors of production, including capital, labor.

Related Questions

4. A rectangle is a four-sided flat shape where every interior angle is a right angle.
Therefore, opposite sides are parallel and consecutive sides are perpendicular.
Rae is drawing rectangle PQRS on a coordinate plane. The rectangle has
coordinates P(-1,2), Q(2,4), R(x, y), and S(3,-4).

Answers

Answer:

coordinates of R(x,y) is R(6,-2).

Step-by-step explanation:

A. we know that

The rectangle has coordinates P (-1,2), Q(2,4), R(x,y) and S(3,-4)

using a graph tool

see the attached figure

Step 1

Find the slope of the line PS

the slope is equal to

m=(y2-y1)/(x2-x1)

mPS=(-4-2)/(3+1)=-6/4=-3/2

Step 2

Find the equation of the line QR

we know that

PS and QR are parallel lines

so

their slopes are the same

mQR=-3/2

with mQR and the point Q(2,4) find the equation of the line

y-y1=m*(x-x1)

y-4=(-3/2)*(x-2)

y=(-3/2)*x+3+4--------> y=7-(3/2)*x

Step 3

Find the equation of the line SR

we know that

SR and QR are perpendicular lines

so

the product of their slopes is equal to minus one

mSR*mQR=-1

mSR=2/3

with mSR and the point S(3,-4) find the equation of the line SR

y-y1=m*(x-x1)

y+4=(2/3)*(x-3)

y=(2/3)*x-2-4---------> y=-6+(2/3)*x

Step 4

Find the intersection of the lines QR and SR

we know that

the intersection of the lines QR and SR is the point R

y=7-(3/2)*x------> equation 1

y=-6+(2/3)*x------> equation 2

equate equation 1 and equation 2

7-(3/2)*x=-6+(2/3)*x

7+6=(2/3)*x+(3/2)*x

13=(2/3)*x+(3/2)*x

Multiply by 6 both sides

6*13=4*x+9*x

13*x=6*13

x=6

find the value of y

y=-6+(2/3)*x------> y=-6+(2/3)*6---------> y=-6+4

y=-2

therefore, the coordinates of point R are (6,-2)

Somebody plz answer just number 15 and 13
Thank you!!

Answers

Answer:

13. 60

15. 200

Step-by-step explanation:

These are the answers because:

1) Since they are asking us to round to the tens place or the hundreds place we have to look at the ones digit and the tens digit.

2) For 13, the answer is 60 because the rounding digit is 5. Since the digit is 5, we have to round up the digit in the tens place. Therefore, it becomes 60.

3) For 15, the answer is 200 because the rounding digit is 3. Since the digit is 3, we have to keep the same number. Therefore, the answer is 200.

Hope this helps!

Find a and d for the function f(x) = a cos(x) + d such that the graph off matches the figure.​

Answers

a = amplitude

d = shift

Just from looking at the graph, I can tell that d = -3.5

The amplitude = 1/2

We want to find the coefficients of a given function such that the graph of the function matches the given graph.

We will get:

a = -0.5

d = -3

------------------------

We start with the general function:

f(x) = a*cos(x) + d.

Let's analyze the graph

In the figure, we can see that the graph passes through the point (0, -3.5)

This means that our function evaluated in x = 0 is equal to -3.5, then we can write:

f(0) = a*cos(0) + d = -3.5

        a + d = -3.5

We also can see that the distance between a peak and a through is 1 unit, and a, the amplitude, must be (in absolute value) half of that. Also note that when x = 0 we have a local minimum, while for the normal cosine we have a local maximum at x = 0, then a must be a negative number.

Then we have that:

a = - (1/2) = -0.5

Now that we know the value of a we can replace it in the other equation to get:

a + d = -3.5

-0.5 + d = -3.5

d = -3.5 + 0.5 = -3

Then the values of a and d are:

a = -0.5

d = -3

The function is:

f(x) = -0.5*cos(x) - 3

And the graph of this can be seen at the end of the answer, where it does not match exactly because the x-scale in the image is in units of pi, while in my graph is in integers.

If you want to learn more, you can read:

https://brainly.com/question/17954123

Find the slope of the line that passes through (-4.5, 7/2 ) and (3, 3.5).


-1/6
0
undefined
-6

Answers

Answer:

The slope of the line is 0 (there is no change in the y-axis)

Step-by-step explanation:

Here, we want to find the slope of the line that passes through the given points

We use a formula for this;

Mathematically;

m = y2-y1/x2-x1

Substituting the values;

m= (3.5-7/2)/3.5+4.5 = 0/8 = 0

The slope of the line is 0

I need the answer to this equation 2x + 8 = 2x -3 as soon as possible doing a tesst

Answers

Ppshdh. Skdjjs. Jshfbn.

-13/20 - 7/5=? Its fractions.

Answers

Answer: 41/20

Step-by-step explanation:

there is a possibility that this is incorrect bb srry

Please help with this math problem:)

Answers

Answer:

See SBS Explanation

Step-by-step explanation:

1. Produces a number that can be classified as an Irrational number.

A:  Square root of 2.

2. Produces a number that an be classified as a Rational number.

A: Square root of 4.

3. Rational number that can be classified as an integer.

A: Negative 2/1 (-2/1)

4. Rational number that produces a repeating decimal.

A: 5/9 (Decimal is 0.5 repeating or 0.555....)

5. Rational number that produces a terminating decimal.

A: 1/4 (Decimal is 0.25)

2p-3 = 3p+2.



answer please!!!!​

Answers

Answer:

- 5

Step-by-step explanation:

Step 1:

2p - 3 = 3p + 2

Step 2:

2p = 3p + 2 + 3

Step 2:

2p = 3p + 5

Step 3:

- p = 5

Answer:

p = - 5

Hope This Helps :)

Answer:

-5

Step-by-step explanation:

2p - 3 = 3p + 2

combine the like terms

-3 -2 = 3p -2p

-5 = p

27. If x = -6, which inequality is true?
A. -5 - 3x > 10
B. -3 - 5x < -14
C. 1 - 2x > 13
D. 2 - X <-3

Answers

Answer:

A. -5 - 3x > 10

Step-by-step explanation:

x = -6

A. -5 - 3(-6) > 10

-5 + 18 = 13 > 10

true

B. -3 - 5(-6) < -14

-3 + 30 = 27 < -14

false

C. 1 - 2(-6) > 13

1 + 12 = 13 > 13

false

D. 2 - (-6) <-3

2 + 6 = 8 < -3

false

Find the slope
x+3=0

Answers

Answer: undefined

You would subtract 3 from both sides to get x= -3 which would be a vertical line meaning it has no slope

can someone plz help me with this

Answers

Answer: Commutative Property of Multiplication/Addition

Step-by-step explanation:

The commutative property is simple

x + y = y + x

or for multiplication

x * y = y * x

what they said above. hope u do well!!! ^^ <3

on a number line, point D is located at 9 and point P is located at -11 what is the distance from d to p

Answers

Answer:

20

Step-by-step explanation:

The distance between two points on the number line is the absolute value of the difference of their coordinates.

coordinate of point D: 9

coordinate of point P: -11

Take the absolute value of their difference. It does not make a difference in which order you subtract the coordinates.

distance = |-11 - 9| = |-20| = 20

Answer: The distance is 20.

A phone company charges a base fee of $12 per month plus an additional charge per minute. The monthly phone cost can be represented by this equation: = 12 + • , where is the additional charge per minute, and is the number of minutes used. Which equation can be used to find the number of minutes a customer used if we know and ? a. = C-12/a b. =(−12)− c. =−12 d. = −12

Answers

Answer:

-12

Step-by-step explanation:

i had got it right yay

Answer:

:)

Step-by-step explanation:

m=c-12/a

I had this on a test and I got it right

The end points of cd are given.find the coordinates of the midpoint M. C(-4,7) and D(0,-3)

Answers

Answer:

M=(-2,2)

Step-by-step explanation:

Xm= -4 + 0/2 --> -4/2

Ym= 7 + -3/2 --> 4/2

Which equation describes a relationship that is directly proportional?

Answers

Answer:

The equation for the proportional relationship is y = 2.8x. If the relationship between two quantities is a proportional relationship, this relationship can be represented by the graph of a straight line through the origin with a slope equal to the unit rate.

Step-by-step explanation:

I hope this helps you :)

17. The volleyball club is raising money for a
new net and equipment. They need $842.36 to
purchase the swing set and have received a
$300.00 donation. The remaining amount will
be equally divided among 4 different
fundraisers throughout the summer. How
much money will each fundraiser need to raise
in order to purchase the new net and
equipment?
$2,169.44
$542.36
0 $135.59
O $210.59

Answers

The answer is $542.36

The money will each fundraiser need to raise in order to purchase the new net and equipment is $135.59. Therefore, option C is the correct answer.

Given that, the volleyball club need $842.36 to purchase the swing set and have received a $300.00 donation.

What is an equation?

In mathematics, an equation is a formula that expresses the equality of two expressions, by connecting them with the equals sign =.

Let the money will each fundraiser need to raise in order to purchase the new net and equipment be x.

Now, the remaining amount = 842.36 - 300.00

= $542.36

The remaining amount will be equally divided among 4 different fundraisers = x = 542.36/4

=135.59

The money will each fundraiser need to raise in order to purchase the new net and equipment is $135.59. Therefore, option C is the correct answer.

To learn more about an equation visit:

brainly.com/question/14686792.

#SPJ5

A group of adults were asked to name their favourite juice. The table gives information about their answers. Work out the angles that would be used to draw a pie chart.

FRUIT JUCE || ORANGE || APPLE || GRAPE || OTHER||

NUMBER || 40 || 27 || 15 || 8 ||

ANGLE || ? || ? || ? || ? ||

Answers

Answer:

ORANGE: 160

APPLE: 108°

GRAPE: 60

OTHER: 32

Step-by-step explanation:

THE SUM OF 40, 27, 15 AND 8 IS 90

90 GOES INTO 360 (THE ANGLES OF A CIRCLE) EXACTLY FOUR TIMES

TO GET EACH ANGLE OF THE PIE CHART YOU SIMPLY NEED TO TIMES EACH NUMBER BY FOUR!

Imagine she found 100 worms of the same length as she found on Tuesday. What would the total length of the 100 worms be? (write the numerical answer only.)

Answers

Step-by-step explanation:

you didn't give us the full question. what were the lengths of the worms found on Tuesday?

Y=2x-1 Y=x+4 Simultaneous equation

Answers

Answer:

(5, 9)

Step-by-step explanation:

Solve by Graphical Method.

I graphed the lines on a graph. The lines intercept at the point (5, 9).

(5, 9) is the solution to the system of equations.

x = 5

y = 9

Hope this helps.

how do you say 19,034,600 in word form?​

Answers

Answer:

nineteen million thirty four thousand six hundred

I think this is right

Step-by-step explanation:

Answer:

Nineteen million thirty four thousand six hundred

Step-by-step explanation:


Find the distance FG between the points
F(0, – 7) and G (-6, -3). Round your answer to the
nearest tenth, if necessary.
FG

Answers

Answer:

7.2

Step-by-step explanation:

The distance of FG is 7.211

What is distance formula?

Distance between two points (x1,y1) and (x2,y2) is given by:

PQ = √( x2 − x1 )² + ( y2 − y1 )². It is known as distance formula.

Given coordinates:

F(0, – 7) and G (-6, -3).

Using distance formula

= √(-6-0)² + (-3 +7)²

=√36+ 16

=√52

=7.211 units

Hence, the distance of FG is 7.211 units

Learn more about distance formula here:

https://brainly.com/question/71956

#SPJ2

the product of two numbers is 12

Answers

the product of 2 numbers is 12, well the answer to that question would be 2 x 6 or 3 x 4

Determine whether each identity is true or false.

Answers

Answer:

TrueTrueFalseFalseTrueFalse

Step-by-step explanation:

3/4+(1/3÷1/6)-(-1/2)=? Please I need this quick!!!

Answers

Answer:

3 and 1/4

Step-by-step explanation:

What is the area of a rectangle
with a length of 5 cm and a width of
4 cm

Answers

Answer:

Area = 20 cm²

Step-by-step explanation:

Area = width * length

area = 5cm * 4cm

area = 20 cm²

Answer:

20 cm

5x4 =20

length times width

What number needs to be added to 5 and 3 so that the ratio of the first number to
the second becomes 3:2?

Answers

4 can be added to 5, and 3 can be added to 3. This should then give you a ratio of 9:6, which reduces to 3:2!

Jaiden is ordering 2.615, 2.65, 2.7, and 2.623 from least to greatest. Here are his steps: (1) Compare and order 2.7 and 2.65 because they have the fewest digits. (2) Compare each digit of the numbers 2.615 and 2.623. * The units digits are equal. * The tenths digits are equal. * 1 < 2 in the hundredths place so 2.615 < 2.623. (3) Combine the two lists into one: 2.7, 2.65, 2.615, 2.623. Which statement describes Jaiden’s error?

Hurry pleaseeee

Answers

Answer: D

Jaiden should have added zeroes to the ends of the shorter decimals to get 2.615 etc

Step-by-step explanation:

The numbers that have the same unit value but different digits in the decimal place can be ordered using place value.

The statement that describes Jaden's error is; Jaiden should have added

zeros to each of the shorter decimals to get 2.615, 2.650, 2.700, and 2.623,

then he should have compared them using place values.

Reasons:

The steps are;

(1) Comparing the digits to select the ones with the least amount of digits

(2) Comparing the digits in the place values of the numbers that equal number of digits

(3) Combining the list of the above comparison into one

Required:

To select the statement that describes Jaiden's error.

Solution:

The place value indicates the multiple of 10 by which the digit is multiplied. The given numbers have the same values in the ones place value, the difference are in the numbers to the right of the decimal point.

The given numbers are;

2.615

2.65

2.7

2.623

The steps to order the numbers from least to greatest;

Add zeros to the shorter decimals such that each number have the same number of digits after the decimal point; 2.615, 2.650, 2.700 2.623

Then

Select the number with the lowest ones; Which gives all the numbers

Then

From the above list, select the number with the lowest tenth, and move them to the left of the list; Which gives the numbers, 2.615, 2.650, 2.623, 2.700

Next

From the above list, select the number with the lowest hundredth, and move them to the left of the list, in increasing order of the hundredth; Which gives the numbers, 2.615, 2.623, 2.650, 2.700

The numbers 2.615, 2.623,2.650, 2.700,  are in the order from least to

greatest.

Therefore Jaiden's error is that Jaiden should have added zeros to each of

the shorter decimals to get 2.615, 2.650, 2.700, and 2.623, then he should

have compared them using place values.

Learn more here:

https://brainly.com/question/25078902

The possible question options are;

The numbers are listed from greatest to listThe two numbers with the most digits should have been compared firstZeros should have been added to the numbers to get, 2.615, 2.065, 2.007, and 2.623. Then the numbers should be compared using place valueHe should have added zeros to each of the shorter decimals to get 2.615, 2.650, 2.700, and 2.623, then he should have compared them using place values.

What is 6/12 in simplist form?

Answers

It’s 1/2 there you go I hope this was helpful

Write a quadratic function in standard form with zeros 12 and -3

Answers

Answer:

x^2+10x-24

Step-by-step explanation:

hope this helps!!!

Describe a sequence of transformations that shows that Polygon A is congruent to Polygon B.

Answers

Answer:

The sequence of transformations that shows that Polygon A is congruent to Polygon B is given as follows;

1) A translation of 8 blocks to the right followed by  2) a translation of 2 blocks down and 3) a rotation of 90 degrees anticlockwise

Step-by-step explanation:

Given that the dimensions and shape of Polygon A and Polygon B are equal, the sequence of transformations that will show their congruency should include a number of rigid transformations, mainly translations and rotations.

Other Questions
I am not an expert in thisLook at the poster of the painting by Schultz. Write a paragraph about the painting and include the following questions.1. What do you see in the painting?2. What kind of lines have been used?3. What is the mood of the painting?4. What colors have been used?How does it make you feel?5. If you get a chance to remake this painting what would you change and why?PLEASE HELP THIS IS FOR TODAY ONLY Find AB if (-9,2) and B(5,-4) whats the answer of 23,869 in long division Finish the quote by Waitzkin: "If we win because we are winners..."Your answer What is most important in Elie's life at the beginning of Night? Book: Night by Elie Wiesel -3y + 8 - 9y + 5z + (-12) - 7z find the length and breadth of a rectangular plot whose area is 660square metre and perimeter is 104metre What animal is BEST adapted to life in a grassland biome? * Drawing and painting materials consist of a pigment and a...Group of answer choicesa. ...solventb. ...Bindingc. ...gum arabicd. ...binder You intend to estimate a population mean with the following sample. 78.4, 85.3, 50, 68.4, 59.4, 73.7, 73.2, 47.4, 40.5 You believe the population is normally distributed. Find the 99.5% confidence interval. The answer must be done as an open-interval accurate to 2 decimal places. =Decide whether the sentence is grammatically CORRECT or INCORRECT as written.Voy a pie cuidadosomente.o correctO incorrect Eliza's backpack weighs 18 and StartFraction 7 over 9 EndFraction pounds with her math book in it. Without her math book, her backpack weighs 14 and StartFraction 7 over 8 EndFraction pounds. How much does Eliza's math book weigh? A cell membrane is very specific about what it allows across. How does t help the cell? Whats 30 times 30 plus 45 minus 456 You left a glass full of water by a window in your house in the morning. Three hours later you walk by the glass, and the water level is significantly lower than it was in the morning. You have made the observation that the water level in the cup is lower. Then, you ask the following question: Why is the water level in the cup lower? What is a possible hypothesis you could make? Which could be used to evaluate the expression Negative 6 (4 and two-thirds)? (Negative 6) (4) + (negative 6) (two-thirds) (Negative 6) (4) times (negative 6) (two-thirds) (Negative 6 + 4) + (Negative 6 + two-thirds) (Negative 6 + 4) times (negative 6 + two-thirds) when Charlemagne took over an area, he forced everyone to become:A CatholicBMuslimFrenchDBuddhist Data concerning Pellegren Corporation's single product appear below: Fixed expenses are $531,000 per month. The company is currently selling 4,000 units per month. The marketing manager would like to cut the selling price by $14 and increase the advertising budget by $35,000 per month. The marketing manager predicts that these two changes would increase monthly sales by 500 units. What should be the overall effect on the company's monthly net operating income of this change? Select one: a. decrease of $18,000 b. increase of $38,000 c. decrease of $38,000 d. increase of $58,000 What are the values of a and b on the number line? When 19346 is divided by a number,the quotient is 841 and the remainder is 3.What is the number?